• Anúncio Global
    Respostas
    Exibições
    Última mensagem

Álgebra Linear (Sistemas Lineares)

Álgebra Linear (Sistemas Lineares)

Mensagempor AboraBR » Sex Ago 24, 2012 23:42

Na questão abaixo deve ser utilizado os teoremas do escalonamento, e através de análise e pequenos cálculos, determinar as condições das letras A e B. Como devo proceder e analisar?

Determine os valores de "m" e "n" para que o sistema:

\left\{\begin{matrix}
x & -3y & +mz & = & n\\ 
2x & -6y & +2z & = & 4
\end{matrix}\right.

a) Tenha Solução.
b) Não Tenha Solução.
AboraBR
Novo Usuário
Novo Usuário
 
Mensagens: 7
Registrado em: Dom Jun 17, 2012 01:42
Formação Escolar: ENSINO MÉDIO
Área/Curso: Engenharia Elétrica
Andamento: cursando

Re: Álgebra Linear (Sistemas Lineares)

Mensagempor MarceloFantini » Sáb Ago 25, 2012 02:07

Como é um sistema com duas equações e três incógnitas, perceba que os coeficientes da segunda equação são todos iguais aos da primeira vezes dois, logo se multiplicarmos a primeira por 2 e subtrairmos teremos (2m-2)z = 2n-4. Simplificando, (m-1)z = n-2.

Agora, vamos analisar: se m=1 e n \neq 2, não haverá solução; se m \neq 1, teremos infinitas soluções.
Futuro MATEMÁTICO
e^{\pi \cdot i} +1 = 0
MarceloFantini
Colaborador Moderador
Colaborador Moderador
 
Mensagens: 3126
Registrado em: Seg Dez 14, 2009 11:41
Formação Escolar: GRADUAÇÃO
Andamento: formado

Re: Álgebra Linear (Sistemas Lineares)

Mensagempor AboraBR » Sáb Ago 25, 2012 16:40

porque n precisa ser diferente de 2 para que não tenha solução?
AboraBR
Novo Usuário
Novo Usuário
 
Mensagens: 7
Registrado em: Dom Jun 17, 2012 01:42
Formação Escolar: ENSINO MÉDIO
Área/Curso: Engenharia Elétrica
Andamento: cursando

Re: Álgebra Linear (Sistemas Lineares)

Mensagempor MarceloFantini » Sáb Ago 25, 2012 16:47

Porque geometricamente são dois planos paralelos (pois tem o mesmo vetor diretor, que é dado pelos coeficientes de x,y,z) com deslocamentos diferentes.
Futuro MATEMÁTICO
e^{\pi \cdot i} +1 = 0
MarceloFantini
Colaborador Moderador
Colaborador Moderador
 
Mensagens: 3126
Registrado em: Seg Dez 14, 2009 11:41
Formação Escolar: GRADUAÇÃO
Andamento: formado

Re: Álgebra Linear (Sistemas Lineares)

Mensagempor AboraBR » Sáb Ago 25, 2012 17:09

Ok, porém, para que o sistema tenha solução, não há nenhuma condição para "n"?
AboraBR
Novo Usuário
Novo Usuário
 
Mensagens: 7
Registrado em: Dom Jun 17, 2012 01:42
Formação Escolar: ENSINO MÉDIO
Área/Curso: Engenharia Elétrica
Andamento: cursando

Re: Álgebra Linear (Sistemas Lineares)

Mensagempor MarceloFantini » Sáb Ago 25, 2012 20:12

Se m \neq 1, não. Pois os vetores normais não serão colineares e portanto os planos não serão paralelos, o que implica que haverá interseção, ou seja, pelo menos uma solução.
Futuro MATEMÁTICO
e^{\pi \cdot i} +1 = 0
MarceloFantini
Colaborador Moderador
Colaborador Moderador
 
Mensagens: 3126
Registrado em: Seg Dez 14, 2009 11:41
Formação Escolar: GRADUAÇÃO
Andamento: formado

Re: Álgebra Linear (Sistemas Lineares)

Mensagempor AboraBR » Sáb Ago 25, 2012 21:01

Show, me ajudou muito, obrigado.
AboraBR
Novo Usuário
Novo Usuário
 
Mensagens: 7
Registrado em: Dom Jun 17, 2012 01:42
Formação Escolar: ENSINO MÉDIO
Área/Curso: Engenharia Elétrica
Andamento: cursando


Voltar para Sistemas de Equações

 



  • Tópicos relacionados
    Respostas
    Exibições
    Última mensagem

Quem está online

Usuários navegando neste fórum: Nenhum usuário registrado e 3 visitantes

 



Assunto: Taxa de variação
Autor: felipe_ad - Ter Jun 29, 2010 19:44

Como resolvo uma questao desse tipo:

Uma usina de britagem produz pó de pedra, que ao ser depositado no solo, forma uma pilha cônica onde a altura é aproximadamente igual a 4/3 do raio da base.
(a) Determinar a razão de variação do volume em relação ao raio da base.
(b) Se o raio da base varia a uma taxa de 20 cm/s, qual a razão de variação do volume quando o raio mede 2 m?

A letra (a) consegui resolver e cheguei no resultado correto de \frac{4\pi{r}^{2}}{3}
Porem, nao consegui chegar a um resultado correto na letra (b). A resposta certa é 1,066\pi

Alguem me ajuda? Agradeço desde já.


Assunto: Taxa de variação
Autor: Elcioschin - Qua Jun 30, 2010 20:47

V = (1/3)*pi*r²*h ----> h = 4r/3

V = (1/3)*pi*r²*(4r/3) ----> V = (4*pi/9)*r³

Derivando:

dV/dr = (4*pi/9)*(3r²) -----> dV/dr = 4pi*r²/3

Para dr = 20 cm/s = 0,2 m/s e R = 2 m ----> dV/0,2 = (4*pi*2²)/3 ----> dV = (3,2/3)*pi ----> dV ~= 1,066*pi m³/s


Assunto: Taxa de variação
Autor: Guill - Ter Fev 21, 2012 21:17

Temos que o volume é dado por:

V = \frac{4\pi}{3}r^2


Temos, portanto, o volume em função do raio. Podemos diferenciar implicitamente ambos os lados da equação em função do tempo, para encontrar as derivadas em função do tempo:

\frac{dV}{dt} = \frac{8\pi.r}{3}.\frac{dr}{dt}


Sabendo que a taxa de variação do raio é 0,2 m/s e que queremos ataxa de variação do volume quando o raio for 2 m:

\frac{dV}{dt} = \frac{8\pi.2}{3}.\frac{2}{10}

\frac{dV}{dt} = \frac{16\pi}{15}